Is $limlimits_{xto x_0}f'(x)=f'(x_0)$?












6












$begingroup$


Let $f$ be a function defined in the open interval $(a,b)$ and let $x_0in(a,b)$. Suppose in addition that $f'(x)$ exists for all $x_0neq xin(a,b)$. Is the following statement true:




If $limlimits_{xto x_0}f'(x)$ exists, then $f'(x_0)$ exists and $limlimits_{xto x_0}f'(x)=f'(x_0)$.




Thanks!










share|cite|improve this question











$endgroup$












  • $begingroup$
    The statement in your question is saying that $f'(x)$ is continuous at some $x_0$. Now you must ask yourself if every derivative is continuous.
    $endgroup$
    – Masacroso
    Jun 20 '16 at 11:36






  • 1




    $begingroup$
    No, this is weaker than saying $f'$ is continuous.
    $endgroup$
    – GEdgar
    Jun 20 '16 at 21:07
















6












$begingroup$


Let $f$ be a function defined in the open interval $(a,b)$ and let $x_0in(a,b)$. Suppose in addition that $f'(x)$ exists for all $x_0neq xin(a,b)$. Is the following statement true:




If $limlimits_{xto x_0}f'(x)$ exists, then $f'(x_0)$ exists and $limlimits_{xto x_0}f'(x)=f'(x_0)$.




Thanks!










share|cite|improve this question











$endgroup$












  • $begingroup$
    The statement in your question is saying that $f'(x)$ is continuous at some $x_0$. Now you must ask yourself if every derivative is continuous.
    $endgroup$
    – Masacroso
    Jun 20 '16 at 11:36






  • 1




    $begingroup$
    No, this is weaker than saying $f'$ is continuous.
    $endgroup$
    – GEdgar
    Jun 20 '16 at 21:07














6












6








6


2



$begingroup$


Let $f$ be a function defined in the open interval $(a,b)$ and let $x_0in(a,b)$. Suppose in addition that $f'(x)$ exists for all $x_0neq xin(a,b)$. Is the following statement true:




If $limlimits_{xto x_0}f'(x)$ exists, then $f'(x_0)$ exists and $limlimits_{xto x_0}f'(x)=f'(x_0)$.




Thanks!










share|cite|improve this question











$endgroup$




Let $f$ be a function defined in the open interval $(a,b)$ and let $x_0in(a,b)$. Suppose in addition that $f'(x)$ exists for all $x_0neq xin(a,b)$. Is the following statement true:




If $limlimits_{xto x_0}f'(x)$ exists, then $f'(x_0)$ exists and $limlimits_{xto x_0}f'(x)=f'(x_0)$.




Thanks!







calculus






share|cite|improve this question















share|cite|improve this question













share|cite|improve this question




share|cite|improve this question








edited Jun 20 '16 at 20:16









snulty

2,86811338




2,86811338










asked Jun 20 '16 at 11:29









boazboaz

2,400614




2,400614












  • $begingroup$
    The statement in your question is saying that $f'(x)$ is continuous at some $x_0$. Now you must ask yourself if every derivative is continuous.
    $endgroup$
    – Masacroso
    Jun 20 '16 at 11:36






  • 1




    $begingroup$
    No, this is weaker than saying $f'$ is continuous.
    $endgroup$
    – GEdgar
    Jun 20 '16 at 21:07


















  • $begingroup$
    The statement in your question is saying that $f'(x)$ is continuous at some $x_0$. Now you must ask yourself if every derivative is continuous.
    $endgroup$
    – Masacroso
    Jun 20 '16 at 11:36






  • 1




    $begingroup$
    No, this is weaker than saying $f'$ is continuous.
    $endgroup$
    – GEdgar
    Jun 20 '16 at 21:07
















$begingroup$
The statement in your question is saying that $f'(x)$ is continuous at some $x_0$. Now you must ask yourself if every derivative is continuous.
$endgroup$
– Masacroso
Jun 20 '16 at 11:36




$begingroup$
The statement in your question is saying that $f'(x)$ is continuous at some $x_0$. Now you must ask yourself if every derivative is continuous.
$endgroup$
– Masacroso
Jun 20 '16 at 11:36




1




1




$begingroup$
No, this is weaker than saying $f'$ is continuous.
$endgroup$
– GEdgar
Jun 20 '16 at 21:07




$begingroup$
No, this is weaker than saying $f'$ is continuous.
$endgroup$
– GEdgar
Jun 20 '16 at 21:07










3 Answers
3






active

oldest

votes


















8












$begingroup$

A qualified "yes": If $f$ is continuous at $x_{0}$, and if $limlimits_{x to x_{0}}f'(x) = L$ exists, then $f$ is differentiable at $x_{0}$, and $f'(x_{0}) = L$.



Qualitatively, the derivative of a continuous function cannot have a removable discontinuity. (If you don't assume $f$ is continuous, then $f$ itself can have a removable or jump discontinuity.)



The claim follows from the Mean Value Theorem: If $delta > 0$ and $f'(x_{0} + h)$ is defined for $0 < |h| < delta$, then for each such $h$, the Mean Value Theorem (applied to $f$ on the closed interval with endpoints $x_{0}$ and $x_{0} + h$) says there is a $t$ between $x_{0}$ and $x_{0} + h$ such that
$$
frac{f(x_{0} + h) - f(x_{0})}{h} = f'(t).
$$
Since $|t - x_{0}| < |h|$, taking the limit as $h to 0$ forces $t - x_{0} to 0$, as well, so
$$
f'(x_{0}) = lim_{h to 0} frac{f(x_{0} + h) - f(x_{0})}{h}
= lim_{t to x_{0}} f'(t).
$$



(Continuity of $f$ was needed to invoke the Mean Value Theorem.)






share|cite|improve this answer











$endgroup$









  • 2




    $begingroup$
    Thanks Andrew, a great answer.
    $endgroup$
    – boaz
    Jun 20 '16 at 12:04










  • $begingroup$
    You're welcome. :) (And, fixed a couple of typos.)
    $endgroup$
    – Andrew D. Hwang
    Jun 20 '16 at 20:03






  • 1




    $begingroup$
    This is a useful fact to know when you do those popular tricky questions like: is $f(x)=x^2sin(x), x ne 0, f(0)=0$ differentiable everywhere?
    $endgroup$
    – GEdgar
    Jun 20 '16 at 21:10












  • $begingroup$
    Since you assume completeness, then you can also get to the same result by using de l'Hôpital's rule to solve the limit. However the statement works also in a non complete field such as Q, I think. Have a look at my my answer, if you like.
    $endgroup$
    – Matteo
    Dec 18 '18 at 9:42





















0












$begingroup$

No, it is not true. For a simple counterexample take the indicator function $chi_{[0,1)}$ in $xin(-1,1)$ and take $x_0$ to be 0.






share|cite|improve this answer









$endgroup$





















    0












    $begingroup$

    Not neccessarily true. We need $f'$ to be continuous at $x_0$. Actually the conditions that $lim_{xto x_0} f'(x)$ and $f'(x_0)$ exist and they are equal to each other are the conditions for the function $f'$ to be continous.



    Here's a counter-example: Consider the function:



    $$f(x) = begin{cases} 0, & mbox{if } mbox{$x le 0$} \ x, & mbox{if } mbox{$x>0$} end{cases}$$



    Obviously $f$ is defined on $mathbb{R}$, but the function isn't differentiable at $0$ and $f'$ is discontinuous at $0$. So by choosing $x_0 = 0$ all the conditions are satisfied, but the result isn't, hence the implication is wrong.






    share|cite|improve this answer











    $endgroup$













    • $begingroup$
      In the proposed example, $f'$ has one-sided limits, but does not have a two-sided limit...?
      $endgroup$
      – Andrew D. Hwang
      Jun 20 '16 at 11:56










    • $begingroup$
      @AndrewD.Hwang Yeah, both one-sided limits exist, but they are not equal to each other, hence the two-sided limit doesn't exists. At the end by checking the graph we can see that it's continuous, therefore both one-sided limits exist, but it's not "smooth" so we expect them not to be equal to each other.
      $endgroup$
      – Stefan4024
      Jun 20 '16 at 11:59






    • 1




      $begingroup$
      Please, change “$f'$ is continuous” into “$f$ is continuous at $x_0$”.
      $endgroup$
      – egreg
      Jun 20 '16 at 21:25






    • 1




      $begingroup$
      This is an incorrect answer to the question. $lim_{xto 0} f'(x)$ does not exist in your example.
      $endgroup$
      – zhw.
      Jun 20 '16 at 21:39











    Your Answer





    StackExchange.ifUsing("editor", function () {
    return StackExchange.using("mathjaxEditing", function () {
    StackExchange.MarkdownEditor.creationCallbacks.add(function (editor, postfix) {
    StackExchange.mathjaxEditing.prepareWmdForMathJax(editor, postfix, [["$", "$"], ["\\(","\\)"]]);
    });
    });
    }, "mathjax-editing");

    StackExchange.ready(function() {
    var channelOptions = {
    tags: "".split(" "),
    id: "69"
    };
    initTagRenderer("".split(" "), "".split(" "), channelOptions);

    StackExchange.using("externalEditor", function() {
    // Have to fire editor after snippets, if snippets enabled
    if (StackExchange.settings.snippets.snippetsEnabled) {
    StackExchange.using("snippets", function() {
    createEditor();
    });
    }
    else {
    createEditor();
    }
    });

    function createEditor() {
    StackExchange.prepareEditor({
    heartbeatType: 'answer',
    autoActivateHeartbeat: false,
    convertImagesToLinks: true,
    noModals: true,
    showLowRepImageUploadWarning: true,
    reputationToPostImages: 10,
    bindNavPrevention: true,
    postfix: "",
    imageUploader: {
    brandingHtml: "Powered by u003ca class="icon-imgur-white" href="https://imgur.com/"u003eu003c/au003e",
    contentPolicyHtml: "User contributions licensed under u003ca href="https://creativecommons.org/licenses/by-sa/3.0/"u003ecc by-sa 3.0 with attribution requiredu003c/au003e u003ca href="https://stackoverflow.com/legal/content-policy"u003e(content policy)u003c/au003e",
    allowUrls: true
    },
    noCode: true, onDemand: true,
    discardSelector: ".discard-answer"
    ,immediatelyShowMarkdownHelp:true
    });


    }
    });














    draft saved

    draft discarded


















    StackExchange.ready(
    function () {
    StackExchange.openid.initPostLogin('.new-post-login', 'https%3a%2f%2fmath.stackexchange.com%2fquestions%2f1833147%2fis-lim-limits-x-to-x-0fx-fx-0%23new-answer', 'question_page');
    }
    );

    Post as a guest















    Required, but never shown

























    3 Answers
    3






    active

    oldest

    votes








    3 Answers
    3






    active

    oldest

    votes









    active

    oldest

    votes






    active

    oldest

    votes









    8












    $begingroup$

    A qualified "yes": If $f$ is continuous at $x_{0}$, and if $limlimits_{x to x_{0}}f'(x) = L$ exists, then $f$ is differentiable at $x_{0}$, and $f'(x_{0}) = L$.



    Qualitatively, the derivative of a continuous function cannot have a removable discontinuity. (If you don't assume $f$ is continuous, then $f$ itself can have a removable or jump discontinuity.)



    The claim follows from the Mean Value Theorem: If $delta > 0$ and $f'(x_{0} + h)$ is defined for $0 < |h| < delta$, then for each such $h$, the Mean Value Theorem (applied to $f$ on the closed interval with endpoints $x_{0}$ and $x_{0} + h$) says there is a $t$ between $x_{0}$ and $x_{0} + h$ such that
    $$
    frac{f(x_{0} + h) - f(x_{0})}{h} = f'(t).
    $$
    Since $|t - x_{0}| < |h|$, taking the limit as $h to 0$ forces $t - x_{0} to 0$, as well, so
    $$
    f'(x_{0}) = lim_{h to 0} frac{f(x_{0} + h) - f(x_{0})}{h}
    = lim_{t to x_{0}} f'(t).
    $$



    (Continuity of $f$ was needed to invoke the Mean Value Theorem.)






    share|cite|improve this answer











    $endgroup$









    • 2




      $begingroup$
      Thanks Andrew, a great answer.
      $endgroup$
      – boaz
      Jun 20 '16 at 12:04










    • $begingroup$
      You're welcome. :) (And, fixed a couple of typos.)
      $endgroup$
      – Andrew D. Hwang
      Jun 20 '16 at 20:03






    • 1




      $begingroup$
      This is a useful fact to know when you do those popular tricky questions like: is $f(x)=x^2sin(x), x ne 0, f(0)=0$ differentiable everywhere?
      $endgroup$
      – GEdgar
      Jun 20 '16 at 21:10












    • $begingroup$
      Since you assume completeness, then you can also get to the same result by using de l'Hôpital's rule to solve the limit. However the statement works also in a non complete field such as Q, I think. Have a look at my my answer, if you like.
      $endgroup$
      – Matteo
      Dec 18 '18 at 9:42


















    8












    $begingroup$

    A qualified "yes": If $f$ is continuous at $x_{0}$, and if $limlimits_{x to x_{0}}f'(x) = L$ exists, then $f$ is differentiable at $x_{0}$, and $f'(x_{0}) = L$.



    Qualitatively, the derivative of a continuous function cannot have a removable discontinuity. (If you don't assume $f$ is continuous, then $f$ itself can have a removable or jump discontinuity.)



    The claim follows from the Mean Value Theorem: If $delta > 0$ and $f'(x_{0} + h)$ is defined for $0 < |h| < delta$, then for each such $h$, the Mean Value Theorem (applied to $f$ on the closed interval with endpoints $x_{0}$ and $x_{0} + h$) says there is a $t$ between $x_{0}$ and $x_{0} + h$ such that
    $$
    frac{f(x_{0} + h) - f(x_{0})}{h} = f'(t).
    $$
    Since $|t - x_{0}| < |h|$, taking the limit as $h to 0$ forces $t - x_{0} to 0$, as well, so
    $$
    f'(x_{0}) = lim_{h to 0} frac{f(x_{0} + h) - f(x_{0})}{h}
    = lim_{t to x_{0}} f'(t).
    $$



    (Continuity of $f$ was needed to invoke the Mean Value Theorem.)






    share|cite|improve this answer











    $endgroup$









    • 2




      $begingroup$
      Thanks Andrew, a great answer.
      $endgroup$
      – boaz
      Jun 20 '16 at 12:04










    • $begingroup$
      You're welcome. :) (And, fixed a couple of typos.)
      $endgroup$
      – Andrew D. Hwang
      Jun 20 '16 at 20:03






    • 1




      $begingroup$
      This is a useful fact to know when you do those popular tricky questions like: is $f(x)=x^2sin(x), x ne 0, f(0)=0$ differentiable everywhere?
      $endgroup$
      – GEdgar
      Jun 20 '16 at 21:10












    • $begingroup$
      Since you assume completeness, then you can also get to the same result by using de l'Hôpital's rule to solve the limit. However the statement works also in a non complete field such as Q, I think. Have a look at my my answer, if you like.
      $endgroup$
      – Matteo
      Dec 18 '18 at 9:42
















    8












    8








    8





    $begingroup$

    A qualified "yes": If $f$ is continuous at $x_{0}$, and if $limlimits_{x to x_{0}}f'(x) = L$ exists, then $f$ is differentiable at $x_{0}$, and $f'(x_{0}) = L$.



    Qualitatively, the derivative of a continuous function cannot have a removable discontinuity. (If you don't assume $f$ is continuous, then $f$ itself can have a removable or jump discontinuity.)



    The claim follows from the Mean Value Theorem: If $delta > 0$ and $f'(x_{0} + h)$ is defined for $0 < |h| < delta$, then for each such $h$, the Mean Value Theorem (applied to $f$ on the closed interval with endpoints $x_{0}$ and $x_{0} + h$) says there is a $t$ between $x_{0}$ and $x_{0} + h$ such that
    $$
    frac{f(x_{0} + h) - f(x_{0})}{h} = f'(t).
    $$
    Since $|t - x_{0}| < |h|$, taking the limit as $h to 0$ forces $t - x_{0} to 0$, as well, so
    $$
    f'(x_{0}) = lim_{h to 0} frac{f(x_{0} + h) - f(x_{0})}{h}
    = lim_{t to x_{0}} f'(t).
    $$



    (Continuity of $f$ was needed to invoke the Mean Value Theorem.)






    share|cite|improve this answer











    $endgroup$



    A qualified "yes": If $f$ is continuous at $x_{0}$, and if $limlimits_{x to x_{0}}f'(x) = L$ exists, then $f$ is differentiable at $x_{0}$, and $f'(x_{0}) = L$.



    Qualitatively, the derivative of a continuous function cannot have a removable discontinuity. (If you don't assume $f$ is continuous, then $f$ itself can have a removable or jump discontinuity.)



    The claim follows from the Mean Value Theorem: If $delta > 0$ and $f'(x_{0} + h)$ is defined for $0 < |h| < delta$, then for each such $h$, the Mean Value Theorem (applied to $f$ on the closed interval with endpoints $x_{0}$ and $x_{0} + h$) says there is a $t$ between $x_{0}$ and $x_{0} + h$ such that
    $$
    frac{f(x_{0} + h) - f(x_{0})}{h} = f'(t).
    $$
    Since $|t - x_{0}| < |h|$, taking the limit as $h to 0$ forces $t - x_{0} to 0$, as well, so
    $$
    f'(x_{0}) = lim_{h to 0} frac{f(x_{0} + h) - f(x_{0})}{h}
    = lim_{t to x_{0}} f'(t).
    $$



    (Continuity of $f$ was needed to invoke the Mean Value Theorem.)







    share|cite|improve this answer














    share|cite|improve this answer



    share|cite|improve this answer








    edited Jun 20 '16 at 20:02

























    answered Jun 20 '16 at 11:53









    Andrew D. HwangAndrew D. Hwang

    53.1k448114




    53.1k448114








    • 2




      $begingroup$
      Thanks Andrew, a great answer.
      $endgroup$
      – boaz
      Jun 20 '16 at 12:04










    • $begingroup$
      You're welcome. :) (And, fixed a couple of typos.)
      $endgroup$
      – Andrew D. Hwang
      Jun 20 '16 at 20:03






    • 1




      $begingroup$
      This is a useful fact to know when you do those popular tricky questions like: is $f(x)=x^2sin(x), x ne 0, f(0)=0$ differentiable everywhere?
      $endgroup$
      – GEdgar
      Jun 20 '16 at 21:10












    • $begingroup$
      Since you assume completeness, then you can also get to the same result by using de l'Hôpital's rule to solve the limit. However the statement works also in a non complete field such as Q, I think. Have a look at my my answer, if you like.
      $endgroup$
      – Matteo
      Dec 18 '18 at 9:42
















    • 2




      $begingroup$
      Thanks Andrew, a great answer.
      $endgroup$
      – boaz
      Jun 20 '16 at 12:04










    • $begingroup$
      You're welcome. :) (And, fixed a couple of typos.)
      $endgroup$
      – Andrew D. Hwang
      Jun 20 '16 at 20:03






    • 1




      $begingroup$
      This is a useful fact to know when you do those popular tricky questions like: is $f(x)=x^2sin(x), x ne 0, f(0)=0$ differentiable everywhere?
      $endgroup$
      – GEdgar
      Jun 20 '16 at 21:10












    • $begingroup$
      Since you assume completeness, then you can also get to the same result by using de l'Hôpital's rule to solve the limit. However the statement works also in a non complete field such as Q, I think. Have a look at my my answer, if you like.
      $endgroup$
      – Matteo
      Dec 18 '18 at 9:42










    2




    2




    $begingroup$
    Thanks Andrew, a great answer.
    $endgroup$
    – boaz
    Jun 20 '16 at 12:04




    $begingroup$
    Thanks Andrew, a great answer.
    $endgroup$
    – boaz
    Jun 20 '16 at 12:04












    $begingroup$
    You're welcome. :) (And, fixed a couple of typos.)
    $endgroup$
    – Andrew D. Hwang
    Jun 20 '16 at 20:03




    $begingroup$
    You're welcome. :) (And, fixed a couple of typos.)
    $endgroup$
    – Andrew D. Hwang
    Jun 20 '16 at 20:03




    1




    1




    $begingroup$
    This is a useful fact to know when you do those popular tricky questions like: is $f(x)=x^2sin(x), x ne 0, f(0)=0$ differentiable everywhere?
    $endgroup$
    – GEdgar
    Jun 20 '16 at 21:10






    $begingroup$
    This is a useful fact to know when you do those popular tricky questions like: is $f(x)=x^2sin(x), x ne 0, f(0)=0$ differentiable everywhere?
    $endgroup$
    – GEdgar
    Jun 20 '16 at 21:10














    $begingroup$
    Since you assume completeness, then you can also get to the same result by using de l'Hôpital's rule to solve the limit. However the statement works also in a non complete field such as Q, I think. Have a look at my my answer, if you like.
    $endgroup$
    – Matteo
    Dec 18 '18 at 9:42






    $begingroup$
    Since you assume completeness, then you can also get to the same result by using de l'Hôpital's rule to solve the limit. However the statement works also in a non complete field such as Q, I think. Have a look at my my answer, if you like.
    $endgroup$
    – Matteo
    Dec 18 '18 at 9:42













    0












    $begingroup$

    No, it is not true. For a simple counterexample take the indicator function $chi_{[0,1)}$ in $xin(-1,1)$ and take $x_0$ to be 0.






    share|cite|improve this answer









    $endgroup$


















      0












      $begingroup$

      No, it is not true. For a simple counterexample take the indicator function $chi_{[0,1)}$ in $xin(-1,1)$ and take $x_0$ to be 0.






      share|cite|improve this answer









      $endgroup$
















        0












        0








        0





        $begingroup$

        No, it is not true. For a simple counterexample take the indicator function $chi_{[0,1)}$ in $xin(-1,1)$ and take $x_0$ to be 0.






        share|cite|improve this answer









        $endgroup$



        No, it is not true. For a simple counterexample take the indicator function $chi_{[0,1)}$ in $xin(-1,1)$ and take $x_0$ to be 0.







        share|cite|improve this answer












        share|cite|improve this answer



        share|cite|improve this answer










        answered Jun 20 '16 at 11:33







        user335721






























            0












            $begingroup$

            Not neccessarily true. We need $f'$ to be continuous at $x_0$. Actually the conditions that $lim_{xto x_0} f'(x)$ and $f'(x_0)$ exist and they are equal to each other are the conditions for the function $f'$ to be continous.



            Here's a counter-example: Consider the function:



            $$f(x) = begin{cases} 0, & mbox{if } mbox{$x le 0$} \ x, & mbox{if } mbox{$x>0$} end{cases}$$



            Obviously $f$ is defined on $mathbb{R}$, but the function isn't differentiable at $0$ and $f'$ is discontinuous at $0$. So by choosing $x_0 = 0$ all the conditions are satisfied, but the result isn't, hence the implication is wrong.






            share|cite|improve this answer











            $endgroup$













            • $begingroup$
              In the proposed example, $f'$ has one-sided limits, but does not have a two-sided limit...?
              $endgroup$
              – Andrew D. Hwang
              Jun 20 '16 at 11:56










            • $begingroup$
              @AndrewD.Hwang Yeah, both one-sided limits exist, but they are not equal to each other, hence the two-sided limit doesn't exists. At the end by checking the graph we can see that it's continuous, therefore both one-sided limits exist, but it's not "smooth" so we expect them not to be equal to each other.
              $endgroup$
              – Stefan4024
              Jun 20 '16 at 11:59






            • 1




              $begingroup$
              Please, change “$f'$ is continuous” into “$f$ is continuous at $x_0$”.
              $endgroup$
              – egreg
              Jun 20 '16 at 21:25






            • 1




              $begingroup$
              This is an incorrect answer to the question. $lim_{xto 0} f'(x)$ does not exist in your example.
              $endgroup$
              – zhw.
              Jun 20 '16 at 21:39
















            0












            $begingroup$

            Not neccessarily true. We need $f'$ to be continuous at $x_0$. Actually the conditions that $lim_{xto x_0} f'(x)$ and $f'(x_0)$ exist and they are equal to each other are the conditions for the function $f'$ to be continous.



            Here's a counter-example: Consider the function:



            $$f(x) = begin{cases} 0, & mbox{if } mbox{$x le 0$} \ x, & mbox{if } mbox{$x>0$} end{cases}$$



            Obviously $f$ is defined on $mathbb{R}$, but the function isn't differentiable at $0$ and $f'$ is discontinuous at $0$. So by choosing $x_0 = 0$ all the conditions are satisfied, but the result isn't, hence the implication is wrong.






            share|cite|improve this answer











            $endgroup$













            • $begingroup$
              In the proposed example, $f'$ has one-sided limits, but does not have a two-sided limit...?
              $endgroup$
              – Andrew D. Hwang
              Jun 20 '16 at 11:56










            • $begingroup$
              @AndrewD.Hwang Yeah, both one-sided limits exist, but they are not equal to each other, hence the two-sided limit doesn't exists. At the end by checking the graph we can see that it's continuous, therefore both one-sided limits exist, but it's not "smooth" so we expect them not to be equal to each other.
              $endgroup$
              – Stefan4024
              Jun 20 '16 at 11:59






            • 1




              $begingroup$
              Please, change “$f'$ is continuous” into “$f$ is continuous at $x_0$”.
              $endgroup$
              – egreg
              Jun 20 '16 at 21:25






            • 1




              $begingroup$
              This is an incorrect answer to the question. $lim_{xto 0} f'(x)$ does not exist in your example.
              $endgroup$
              – zhw.
              Jun 20 '16 at 21:39














            0












            0








            0





            $begingroup$

            Not neccessarily true. We need $f'$ to be continuous at $x_0$. Actually the conditions that $lim_{xto x_0} f'(x)$ and $f'(x_0)$ exist and they are equal to each other are the conditions for the function $f'$ to be continous.



            Here's a counter-example: Consider the function:



            $$f(x) = begin{cases} 0, & mbox{if } mbox{$x le 0$} \ x, & mbox{if } mbox{$x>0$} end{cases}$$



            Obviously $f$ is defined on $mathbb{R}$, but the function isn't differentiable at $0$ and $f'$ is discontinuous at $0$. So by choosing $x_0 = 0$ all the conditions are satisfied, but the result isn't, hence the implication is wrong.






            share|cite|improve this answer











            $endgroup$



            Not neccessarily true. We need $f'$ to be continuous at $x_0$. Actually the conditions that $lim_{xto x_0} f'(x)$ and $f'(x_0)$ exist and they are equal to each other are the conditions for the function $f'$ to be continous.



            Here's a counter-example: Consider the function:



            $$f(x) = begin{cases} 0, & mbox{if } mbox{$x le 0$} \ x, & mbox{if } mbox{$x>0$} end{cases}$$



            Obviously $f$ is defined on $mathbb{R}$, but the function isn't differentiable at $0$ and $f'$ is discontinuous at $0$. So by choosing $x_0 = 0$ all the conditions are satisfied, but the result isn't, hence the implication is wrong.







            share|cite|improve this answer














            share|cite|improve this answer



            share|cite|improve this answer








            edited Jun 20 '16 at 21:26

























            answered Jun 20 '16 at 11:35









            Stefan4024Stefan4024

            30.6k63479




            30.6k63479












            • $begingroup$
              In the proposed example, $f'$ has one-sided limits, but does not have a two-sided limit...?
              $endgroup$
              – Andrew D. Hwang
              Jun 20 '16 at 11:56










            • $begingroup$
              @AndrewD.Hwang Yeah, both one-sided limits exist, but they are not equal to each other, hence the two-sided limit doesn't exists. At the end by checking the graph we can see that it's continuous, therefore both one-sided limits exist, but it's not "smooth" so we expect them not to be equal to each other.
              $endgroup$
              – Stefan4024
              Jun 20 '16 at 11:59






            • 1




              $begingroup$
              Please, change “$f'$ is continuous” into “$f$ is continuous at $x_0$”.
              $endgroup$
              – egreg
              Jun 20 '16 at 21:25






            • 1




              $begingroup$
              This is an incorrect answer to the question. $lim_{xto 0} f'(x)$ does not exist in your example.
              $endgroup$
              – zhw.
              Jun 20 '16 at 21:39


















            • $begingroup$
              In the proposed example, $f'$ has one-sided limits, but does not have a two-sided limit...?
              $endgroup$
              – Andrew D. Hwang
              Jun 20 '16 at 11:56










            • $begingroup$
              @AndrewD.Hwang Yeah, both one-sided limits exist, but they are not equal to each other, hence the two-sided limit doesn't exists. At the end by checking the graph we can see that it's continuous, therefore both one-sided limits exist, but it's not "smooth" so we expect them not to be equal to each other.
              $endgroup$
              – Stefan4024
              Jun 20 '16 at 11:59






            • 1




              $begingroup$
              Please, change “$f'$ is continuous” into “$f$ is continuous at $x_0$”.
              $endgroup$
              – egreg
              Jun 20 '16 at 21:25






            • 1




              $begingroup$
              This is an incorrect answer to the question. $lim_{xto 0} f'(x)$ does not exist in your example.
              $endgroup$
              – zhw.
              Jun 20 '16 at 21:39
















            $begingroup$
            In the proposed example, $f'$ has one-sided limits, but does not have a two-sided limit...?
            $endgroup$
            – Andrew D. Hwang
            Jun 20 '16 at 11:56




            $begingroup$
            In the proposed example, $f'$ has one-sided limits, but does not have a two-sided limit...?
            $endgroup$
            – Andrew D. Hwang
            Jun 20 '16 at 11:56












            $begingroup$
            @AndrewD.Hwang Yeah, both one-sided limits exist, but they are not equal to each other, hence the two-sided limit doesn't exists. At the end by checking the graph we can see that it's continuous, therefore both one-sided limits exist, but it's not "smooth" so we expect them not to be equal to each other.
            $endgroup$
            – Stefan4024
            Jun 20 '16 at 11:59




            $begingroup$
            @AndrewD.Hwang Yeah, both one-sided limits exist, but they are not equal to each other, hence the two-sided limit doesn't exists. At the end by checking the graph we can see that it's continuous, therefore both one-sided limits exist, but it's not "smooth" so we expect them not to be equal to each other.
            $endgroup$
            – Stefan4024
            Jun 20 '16 at 11:59




            1




            1




            $begingroup$
            Please, change “$f'$ is continuous” into “$f$ is continuous at $x_0$”.
            $endgroup$
            – egreg
            Jun 20 '16 at 21:25




            $begingroup$
            Please, change “$f'$ is continuous” into “$f$ is continuous at $x_0$”.
            $endgroup$
            – egreg
            Jun 20 '16 at 21:25




            1




            1




            $begingroup$
            This is an incorrect answer to the question. $lim_{xto 0} f'(x)$ does not exist in your example.
            $endgroup$
            – zhw.
            Jun 20 '16 at 21:39




            $begingroup$
            This is an incorrect answer to the question. $lim_{xto 0} f'(x)$ does not exist in your example.
            $endgroup$
            – zhw.
            Jun 20 '16 at 21:39


















            draft saved

            draft discarded




















































            Thanks for contributing an answer to Mathematics Stack Exchange!


            • Please be sure to answer the question. Provide details and share your research!

            But avoid



            • Asking for help, clarification, or responding to other answers.

            • Making statements based on opinion; back them up with references or personal experience.


            Use MathJax to format equations. MathJax reference.


            To learn more, see our tips on writing great answers.




            draft saved


            draft discarded














            StackExchange.ready(
            function () {
            StackExchange.openid.initPostLogin('.new-post-login', 'https%3a%2f%2fmath.stackexchange.com%2fquestions%2f1833147%2fis-lim-limits-x-to-x-0fx-fx-0%23new-answer', 'question_page');
            }
            );

            Post as a guest















            Required, but never shown





















































            Required, but never shown














            Required, but never shown












            Required, but never shown







            Required, but never shown

































            Required, but never shown














            Required, but never shown












            Required, but never shown







            Required, but never shown







            Popular posts from this blog

            Plaza Victoria

            In PowerPoint, is there a keyboard shortcut for bulleted / numbered list?

            How to put 3 figures in Latex with 2 figures side by side and 1 below these side by side images but in...